LSAT and Law School Admissions Forum

Get expert LSAT preparation and law school admissions advice from PowerScore Test Preparation.

 Administrator
PowerScore Staff
  • PowerScore Staff
  • Posts: 8917
  • Joined: Feb 02, 2011
|
#41351
Complete Question Explanation
(The complete setup for this game can be found here: lsat/viewtopic.php?t=13965)

The correct answer choice is (B)

If N is chosen for stop 5, from our template we know that N cannot be chosen for stop 1, and this information eliminates answer choice (E). The template also indicates that O must be chosen for stop 2, and this fact eliminates answer choices (A), (C), and (D). Thus, answer choice (B) is correct. Please note that there are other reasons to eliminate some of the answers, such as the fact that answer choices (A) and (C) violate the fourth rule.

Get the most out of your LSAT Prep Plus subscription.

Analyze and track your performance with our Testing and Analytics Package.